Menos que orden árbol

En QFT es muy típico hacer una primera estimación a “orden árbol”, sin considerar creación de pares particula/antiparticula. Pero particularmente me resulta muy educativo considerar las diferentes fuerzas muy a la antigua, como si estuvieramos en Bohr-Sommerfeld, o incluso en Bohr a secas: considerar la vieja condicion de equilibrio de fuerzas en una orbita circular:

\({V^2 \over r} = {F / m}\)

Moviendo la masa orbitante al lado izquierdo, podemos poner la formula en función del momento angular, y por tanto comparar directamente con la acción minima de mecanica cuántica, la cte de Planck \(h\) (y algunos factores multiplos de pi que siempre me olvido, y que me voy a ahorrar para este comentario)

\(L {V \over r^2} = F\)

El primer caso de interes es el electromagnetismo, o simplemente un campo electrostatico en tres dimensiones espaciales, poniendo \(F = K / r^2\).  El dato del radio se simplifica a uno y otro lado y nos queda

\(L={K\over V}\)

Ahora ponemos un poco de ciencia moderna: por un lado V no puede rebasar la velocidad de la luz, y por otro no puede ser mayor que la cte de Planck. Estas dos condiciones

\(L < {K \over c}; \;\) \(h < L\)

Son las que definen la cte de estructura fina, \(\alpha \approx {K\over c h}\), que cobra su sentido en la zona cercana a la  “saturacion” de las desigualdades, cuando el sistema es a la vez relativista y cuantico. El hecho de que la cte de estructura fina sea pequeña implica que el regimen relativista esta bien regido por la mecanica cuantica, tenemos “autorizacion clasica” para ir a momentos angulares hasta 137 veces más pequeños que la cte de Planck, y es logico que la cuántica tenga algo que decir, pues tiene que prohibir esos nivels. Por cierto, que si calcularamos el radio en función de la velocidad, veriamos que a velocidad c corresponde el radio cero.

Si la constante de estructura fina fuera de orden unidad o mayor, entonces ya no esta tan claro que tengamos algo que hacer con la mecanica cuántica: simplemente las reglas del electromagnetismo dejarian las orbitas estables fuera del dominio de la teoria de Bohr-Sommerfeld.

Otra fuerza interesante es la fuerza constante que se supone tienen los potenciales confinantes como el de QCD. En tal caso, no hay forma de simplificar los radios, y la condicion cuantica parece ser (con las unidades adecuadas para la cte de fuerza, K).

\(h< L < {K r^2 \over c}\)

Pero la condicion “relativista” es mas intriguante. El radio depende de la velocidad, a partir de la ecuacion inicial v^2/r=K/m, o lo que es lo mismo

\(r={m v^2 \over K}\)

asi que en realidad

\(L={K\over v}({m v^2\over K})^2= {m^2 \over K} v^3 = \)

Y en el limite relativista tambien el momento angular tiene un máximo, no un minimo (y de hecho se va a infinito). Nuestra teoria juguete “de cuerdas de Bohr-Sommerfeld” dice que

\(h<{m^2\over K}v^3<{m^2\over K}c^3\)

Y si la particula orbitante tiene masa en reposo distinta de cero, aplicamos \(m=m_0/\sqrt{1-v^2/c^2}\)

\(h<{m_0^2\over K}{v^3\over 1-v^2/c^2}<\infty\)

El resultado es llamativo. Para una masa en reposo lo suficientemente grande, los estados ligados cuanticos ni siquiera son relativistas. La cuerda cuantica relativista cobra mas sentido cuando la particula en el extremo es massless, sin masa en reposo, y entonces v=c. Dejamos como “ejercicio” el considerar qué pasa en cada caso con el radio del estado “fundamental” cuantico, esto es, el de momento angular L=h.

 

 

Leave a Reply

Your email address will not be published. Required fields are marked *

This site uses Akismet to reduce spam. Learn how your comment data is processed.